solve equation show all steps what is 2x-3x+5=18

Answers

Answer 1

Answer:

x = -13

Step-by-step explanation:

2x-3x+5=18

Combine like terms

-x +5 = 18

Subtract 5 from each side

-x +5-5 = 18-5

-x = 13

Multiply each side by -1

x = -13

Answer 2

Answer:

[tex]\huge \boxed{{x=-13}}[/tex]

Step-by-step explanation:

[tex]2x-3x+5=18[/tex]

[tex]\sf Combine \ like \ terms.[/tex]

[tex]-1x+5=18[/tex]

[tex]\sf Subtract \ 5 \ from \ both \ sides.[/tex]

[tex]-1x+5-5=18-5[/tex]

[tex]-1x=13[/tex]

[tex]\sf Multiply \ both \ sides \ by \ -1.[/tex]

[tex]-1x \times (-1)=13 \times (-1)[/tex]

[tex]x=-13[/tex]


Related Questions

50 POINTSS!! While preparing a roof, patty drops a screwdriver from a height of 80 feet. The function (h)t = -16t^2 + 80 gives the height of the screwdriver in the feet as feet after t seconds during its fall. Which of these is the graph of the function

Answers

Answer:

Graph D

Step-by-step explanation:

(h)t = -16t^2 + 80

At time t = 0 the screwdriver is at 0+80 =80 ft

As time increases the height will decrease so we can eliminate graph B

We know this is a downward parabola by the - in front of the t^2 so we can eliminate graph A

We need to find where it meets the x axis

0 = -16t^2 + 80

-80 = -16t^2

5 = t^2

t = sqrt(5)

t =2.23

This is Graph D

Pete earns graduated commission on his sales each month. He earns 7% commission on the first $35,000 in sales and 9% on anything over that. If Pete had $43,000 in sales this month, how much commission did he earn?
a. $2,610
b. $3,170
c. $3,870
d. $6,880
Please select the best answer from the choices provided A B C D

Answers

Answer:

b.$3170

Step-by-step explanation:

43,000-35,000=8,000

SO you would find 9% commission on 8,000

8,000x.09=720

then, you would find his normal commission if he only made 35,000

so you would do 35,000x.07=2450

you would, then, add the two together to get his entire commission, which is $3170

Answer:

b

Step-by-step explanation:

The National Weather Service collects data on the number of hours of consecutive rainfall and the number of minor traffic accidents in a particular city. The scatter plot shows the data it gathered and the line of best fit. For a school project, Peyton uses technology to calculate the equation for line of best fit. If Peyton's calculation is correct, which equation could represent the line of best fit for this data?

Answers

The equation that could best represent the line of best fit for the given data is; y = 0.625x

How to find a linear equation from a scatter plot?

The formula for the equation of a line in slope intercept form is;

y = mx + c

where;

m is slope

c is y-intercept which is the point where the line intersects the y-axis

Now, in this question, we see that the line intersects the y-axis at 0. Thus;

y-intercept; c = 0

Now, let us find the slope from the formula;

m = (y₂ - y₁)/(x₂ - x₁)

Using the first and penultimate coordinate which are;

(0, 0) and (8, 5), we have;

m = (5 - 0)/(8 - 0)

m = 0.625

Thus, the equation is;

y = 0.625x

Read more about Scatter plot equation at; https://brainly.com/question/6592115

#SPJ1

Answer:

y=0.625x

Step-by-step explanation:

plato

100 2/3 as a decimal

Answers

Answer:

100.66 is the decimal.

Step-by-step explanation:

100⅔ = 302/3

302 ÷ 3 = 100.66 to infinity.

A result of a number, what decreased by 15% equal 161. Find that number.

Answers

Answer: 189.41

Step-by-step explanation: Let’s take the number as n and if nx(100-15)/100=161 then 85n/100=161 and therefore n= 181.4117

I don’t know why the answer is in decimal as mostly it won’t be. Maybe you have made a mistake with the question so pls do check the question too!


WikiTongues has
videos of 500 different
languages from around the
world. What percent of the
7,111 total languages
spoken globally
is represented on
WikiTongues?

Answers

Hey there! I'm happy to help!

We see that 500 languages is a certain percent of the 7,111 total languages. When working with percent equations, the word "is" means "equals", and "of" usually means "multiplied by" This means that 500 is equal to a certain percent multiplied by 7,111. We can use p to represent this certain percent and create an equation below.

500=7,111p

Let's flip the equation around so the p is on the left side. Variables are usually supposed to be on the left.

7,111p=500

We divide both sides by 7,111.

p=0.0703

As a percent, this is about 7.03%. Therefore, about 7.03% of the total languages spoken globally are represented on WikiTongues.

I hope that this helps! Have a wonderful day! :D

Answer:

7.03 %

Step-by-step explanation:

500 / 7,111 = 0.07031359865

0.07031359865  x 100 = 7.03 %

Please help correct answer gets brainliest.

Answers

Answer:

C. The charge of each minute of a call is $0.40

Step-by-step explanation:

We look at the point (1, 0.4)

We know that our x = 1 is 1 minute of the call.

We know that y = 0.4 is the cost when 1 minute of the call has elapsed.

Therefore, our answer is C.

Answer:

Step-by-step explanation:

The answer is (c)

The price goes up $0.40 every minute Greg is on a call.

Write an equation for a line on the graph that passes through the points (0.4) and (12,16)

Answers

Answer:

[tex] y = x + 4 [/tex]

Step-by-step explanation:

Use the two-point form of the equation of a line.

[tex] y - y_1 = \dfrac{y_2 - y_1}{x_2 - x_1}(x - x_1) [/tex]

[tex] y - 4 = \dfrac{16 - 4}{12 - 0}(x - 0) [/tex]

[tex] y - 4 = \dfrac{12}{12}x [/tex]

[tex] y - 4 = x [/tex]

[tex] y = x + 4 [/tex]

Answer:

y = x + 4

Step-by-step explanation:

An equation for a line looks like:

=> y = mx +b

=> In this equation "m" is the slope.

=> "b" is the y-intercept.

To find the slope:

=> y/x - y1/x1

=> 16/12 - 4/0

=> 16 -4 / 12 - 0

=> 12 / 12

=> 1

So, the slope is 1.

Now our equation looks like:

y = 1x + b

=> y = x + b

Let's take some the values of "x" and "y"  of (0,4)

So, our now look like:

=> 4 = 1 (0) + b

=> 4 = b

So, b (y-intercept) = 4

Now, our final equation is:

=> y = x + 4

whats the factored form of 6x 2 - 8x - 8 = 0

Answers

Answer:

x = -2/3 , 2

Step-by-step explanation:

Factor 2  out of   6 x^ 2 − 8 x − 8

2 ( 3 x^ 2 − 4 x − 4 ) = 0

Factor

2 ( 3 x + 2 ) ( x − 2 ) = 0

Set  3 x + 2  equal to  0  and solve for  x

x = -2/3

Set  x − 2  equal to  0  and solve for  x

x = 2

The final solution is all the values that make 2 ( 3 x + 2 ) ( x − 2 ) = 0

x = -2/3 , 2

Hope this can help you

 

A. (-2,-1)
B. (-1,-2)
C. (1.-2)
D. (2,-1)

Answers

Answer:

D. 2,-1

Step-by-step explanation:

When a line is reflected by y=x, the x and y switch.

Please answer this question now

Answers

Answer:

72°

Step-by-step explanation:

From the figure given, angle D intercepts arc ABC. According to the Inscribed Angle Theorem:

m < D = ½(ABC) = ½(AB + BC)

Thus,

[tex] 56 = \frac{1}{2}(AB + 40) [/tex]

Solve for AB

[tex] 56 = \frac{AB + 40}{2} [/tex]

Multiply both sides by 2

[tex] 56*2 = \frac{AB + 40}{2}*2 [/tex]

[tex] 112 = AB + 40 [/tex]

Subtract both sides by 40

[tex] 112 - 40 = AB + 40 - 40 [/tex]

[tex] 72 = AB [/tex]

Arc AB = 72°

-3x+2y=6 Find the intercepts. Show your work.

Answers

Answer:

The x-intercept is (-2,0)

The y-intercept is (0,3)

Step-by-step explanation:

An x-intercept is the point when the graph crosses the x-axis. In other words, the y-coordinate of the x-intercept is 0 (since it lays on the x-axis). In other words, to solve for the x-intercept(s), plug in 0 for y and solve for x:

[tex]-3x+2y=6\\-3x+2(0)=6\\-3x=6\\x=-2[/tex]

So, the x-intercept is (-2,0).

Likewise, the y-intercept is the point when the graph crosses the y-axis. Because it's on the y-axis, the x-coordinate value would be 0. Thus, to find the y-intercept, plug in 0 for x and solve for y:

[tex]-3x+2y=6\\-3(0)+2y=6\\2y=6\\y=3[/tex]

Thus, the y-intercept is (0,3).

ASAP!!! PLEASE help me solve this question! No nonsense answers, and solve with full solutions.

Answers

Answer:

When two inscribed angles in one circle both equal 75°, the two angles must intercept the same arc that measures 75°

Step-by-step explanation:

Based on the Inscribed Angles Theorem, the measure of an intercepted arc is twice the measure of the inscribed angle that intercepts it in a circle.

Consequently, the theorem also Holds that the measure of two inscribed angles intercepting the same arc, are congruent. In other words, both angles together are the same, and their sum would give you the measure of the arc they both intercept.

In the diagram shown in the attachment below, we have 2 inscribed angles, angle A and B. They both intercept the same arc of 75°.

Therefore, we can conclude that the measure of both angles equal 75°, which is the same as the measure of the arc they intercept.

Angle A = Angle B

m<A + m<B = measure of intercepted arc = 75°

solve for x 3(x+2) = 12

Answers

Answer:

x=2

Step-by-step explanation:

3(x+2) = 12

Divide by 3

3/3(x+2) = 12/3

x+2 = 4

Subtract 2 from each side

x+2-2 = 4-2

x =2

Answer:

The value of x is equal to 2.

Step-by-step explanation:

3(x + 2) = 12

Distribute 3 to (x + 2)

3x + 6 = 12

Subtract 6 from both sides of the equation.

3x = 6

Divide 3 on both sides of the equation.

x = 2

The value of x is 2

this graph shows the solution to which inequality (3 3) (-3 -1)

Answers

Answer:

B

Step-by-step explanation:

since the graph shows dotted line the sign has to be < or > so C and D eliminated from your answer

y>2/3 x +1 is your answer (B)

Answer:

[tex] \boxed{y > \frac{2}{3} x + 1}[/tex]

Option B is the correct option.

Step-by-step explanation:

point ₁ ( - 3 , - 1 ) x₁ = - 3 , y₂ = - 1

point ₂ ( 3 , 3 ). x₂ = 3 , y₂= 3

Now, let's find the slope:

Slope ( m ) = [tex] = \frac{y2 - y1}{x2 - x1} [/tex]

[tex] = \frac{3 - ( - 1)}{3 - ( - 3)} [/tex]

[tex] = \frac{3 + 1}{3 + 3} [/tex]

[tex] = \frac{4}{6} [/tex]

[tex] = \frac{2}{3} [/tex]

At ( 3 , - 1 )

y = mx + c , where m is the gradient / slope amd c is called the intercept on y-axis

[tex] - 1 = \frac{2}{3} \times ( - 3) + c[/tex]

Solve for c

[tex] - 1 + 2 = c[/tex]

[tex]c = 1[/tex]

Since, The red line is dotted line. Therefore it does not include the equal to part. And the shaded region is upper part. Hence, ' > ' should be used.

The answer would be:

[tex]y > \frac{2}{3} x + 1[/tex]

Hope I helped!

Best regards!

If each of the 4 shelves holds 14 movies, how many movies does Betty have on her shelving unit? Show your work.

Answers

Answer:

62

Step-by-step explanation:

4x + 6

4(14) + 6 (substituting x = 14)

56 + 6

62

MATH QUESTION : As punishment for bad behavior during recess, Mrs. Busywork asked her class to multiply 10 by 1/3 five times. John, however, notices that it is possible to multiply 10 by a single fraction and still get the same answer as the other students. What is this single fraction?

Answers

Answer:

5/3

Step-by-step explanation:

5×(10×1/3)

=10×5/3

=since 5×(10×1/3) gives 50/3 so thus 10×5/3.

The fraction is therefore 5/3

prove that [tex]2/\sqrt{3} cosx + sin x= sec(\pi /6-x)[/tex]

Answers

Answer:

sec(π/6 - x) = R.H.S = 2/(√3cosx + sinx)  = L.H.S.

Step-by-step explanation:

sec(π/6 - x) = 1/cos(π/6 - x)

Using compound angle formula,

cos(A - B) = cosAcosB + sinAsinB where A = π/6 and B = x.

So, cos(π/6 - x) = cos(π/6)cosx + sin(π/6)sinx , cosπ/6 = √3/2 and sinπ/6 = 1/2

cos(π/6 - x) = cosπ/6cosx + sinπ/6sinx = (√3/2)cosx + (1/2)sinx

sec(π/6 - x) = 1/cos(π/6 - x)

= 1/(√3/2)cosx + (1/2)sinx = 2/(√3cosx + sinx)

= L.H.S

So, sec(π/6 - x) = R.H.S = 2/(√3cosx + sinx)  = L.H.S

Convert each number to scientific notation or standard notation.


1.0395 × 10^9

4 × 10^2

Answers

Answer:

1)1,0395x10^9

2)its already in scientific notation?

If you write it in standard notation, which is actually just regular notations it's 400.

Step-by-step explanation:

Scientific notation can be used to represent very large or small numbers. A number written in scientific notation will have one non-zero digit to the left of the decimal point, multiplied by a power of  10 , with the power corresponding to the amount of places the decimal should move.

A. 19
B. 20
C. 15
D. 25
Help please !!

Answers

Answer:

A

Step-by-step explanation:

A survey of 100 people found that 35 people exercise in
the morning, 45 people exercise in the afternoon, and 20
people exercise at night. Tara claims that 35:55, 45:80,
and 20:65 are possible ratios from this data. Jim claims
that 35:100, 45:100, and 20:100 are possible ratios from
this data. Who is correct? Explain.​

Answers

Answer:

Jim's claim is correct

Step-by-step explanation:

Total survey=100 people

Morning exercise=35 people

Afternoon exercise=45 people

Night exercise=20 people

Morning exercise : Total survey

=35 : 100

Afternoon exercise : Total survey

=45 : 100

Night exercise : Total survey

=20 : 100

Jim's claim is correct because in finding the ratio, one has to relate it with the total people involved in the survey.

Tara's claim however seems to relate one variable with two other variables

Example:

35 : 55

Morning exercise : (morning exercise + night exercise)

35 : (35+20)

35 : 55

Answer:

Sample Response: Jim is correct because 35:100, 45:100, and 20:100 are part-to-whole comparisons. There are other possible part-to-part ratios from the data.

Question 13 of 25
Which of the following is a correct name for a side of the given angle?
D
E
A. Point E
B. ED
C. DE
D. Point D

Answers

The answer to this question is D

ASAP!!!!!!!!! PLEASE help me with this question! This is really urgent! No nonsense answers please.

Answers

Answer: C) 63

=================================================

Explanation:

Arcs CBH and FGH are given, while arc CDF is unknown. Let's call this y

y = measure of arc CDF

Adding the three arcs forms a full circle of 360 degrees

(arc CBH)+(arc FGH)+(arc CDF) = 360

170+64+y = 360

y+234 = 360

y = 360-234

y = 126

arc CDF = 126 degrees

Then notice how inscribed angle x cuts off arc CDF. By the inscribed angle theorem, we take half of the arc measure to get the inscribed angle measure.

inscribed angle = (arc measure)/2

x = (arc CDF)/2

x = 126/2

x = 63

Answer:

rewrite the fromula 126

Step-by-step explanation:

Please help, I don't understand

Answers

Answer:

  sin(θ) = (2/5)√6

Step-by-step explanation:

The sine and cosine are related by the formula ...

  [tex]\sin{(\theta)}^2+\cos{(\theta)}^2=1\\\\\sin{(\theta)}=\pm\sqrt{1-\cos{(\theta)}^2}[/tex]

Filling in the given value for cos(θ), we find the sine to be ...

  [tex]\sin{(\theta)}=\pm\sqrt{1-\left(\dfrac{1}{5}\right)^2}=\pm\dfrac{\sqrt{24}}{5}=\pm\dfrac{2}{5}\sqrt{6}[/tex]

__

The cosine function is positive for angles in both the first and fourth quadrants. The restriction on θ tells us this is a first-quadrant angle. The sine is positive in the first quadrant, so the desired value is ...

  sin(θ) = (2/5)√6

24 is 75% percent of what number?

Answers

Answer:

32

Step-by-step explanation:

We can call the number x. 75% can be expressed as 0.75 so we can write the following equation: 0.75x = 24. Dividing both sides by 0.75 to get rid of the coefficient of x, we get that x = 32.

Answer:

32.

Step-by-step explanation:

Let's say the number we are trying to find is x.

24 = 0.75x

0.75x = 24

x = 24 / 0.75

x = 32

Hope this helps!

It is estimated that 52% of drivers text while driving . What is the probability the next driver texting while driving that a police officer pulls over is the fifth driver?

Answers

Answer:

0.0276

Step-by-step explanation:

The computation of the probability that the next driver is texting while driving is shown below:

= Fourth pull failure × fifth pull success

where,

Fourth pull failure is (1 - 0.52)^4

And, the fifth pull success is 0.52

Now placing these values to the above formula

So,

= (1 - 0.52)^4 × 0.52

= 0.0276

Hence, the probability is 0.0276

Plz HELP !!!!! I would appreciate it!!!

Answers

Answer/Step-by-step explanation:

a. The sides of ∆ABC can be labelled as follows, taking θ as the reference point.

Adjacent = side AB

Opposite = side BC

Hypotenuse = side AC

b. I. "<B is an acute angle" => FALSE. It is a right angle (90°)

II. "The side opposite to <C is BC" => FALSE. The side opposite to <C is AB.

III. The side adjacent to <A is AB => TRUE

IV. The side opposite to <B is not the longest. => FALSE. It is the hypotenuse and the longest side.

c. Cos θ = [tex] \frac{adjacent}{hypotenuse} = \frac{AB}{AC} [/tex]

Tan θ = [tex] \frac{opposite}{adjacent} = \frac{BC}{AB} [/tex]

d. Sin C = [tex] \frac{opp}{hypo} = \frac{AB}{AC} [/tex]

Cos C = [tex] \frac{adjacent}{hypotenuse} = \frac{BC}{AC} [/tex]

Tan C = [tex] \frac{opp}{adjacent} = \frac{AB}{BC} [/tex]

Dante is baking two different recipes, cookies and brownies. The cookie recipe requires 1.5 cups of sugar, and the brownie recipe requires 1.25 cups of sugar. Write an addition equation to represent the total amount of sugar Dante needs. Enter your answer as an addition equation, like this: 42+(-53)=-11

Answers

Answer:

2.75 cups of sugar

Step-by-step explanation:

Hello!

To find the total amount of sugar Dante needs we need to add the amount need for each recipe

Cookie recipe needs 1.5 cups

Brownie recipe needs 1.25 cups

1.5 + 1.25 = 2.75 cups of sugar

The answer is 2.75 cups of sugar

Hope this helps!

Answer:

the answer is 1.5+1.25=1.75

Step-by-step explanation:

i just added them together

your welcome ( it also depends on how many times he uses the recipes)

Tres amigos quieren abrir una empresa dedicada al reciclaje de plasticos. Para eso deben contar con $72.000. Uno de los amigos aporto 1/3 de dinero necesario para abrirla, el segundo aporto una cantidad equivalente a los 5/6 del resto.¿cuanto dinero aporto el tercer amigo?¿Cual de los tres hizo el mayor aporte?¿cuanto mas puso en relación con que el que aporto menos?

Answers

Answer:

a) ¿cuanto dinero aporto el tercer amigo?

= $ 8,000

b) ¿Cual de los tres hizo el mayor aporte?

El segundo amigo hizo la mayor contribución.

c)¿cuanto mas puso en relación con que el que aporto menos?

El tercer amigo hizo la menor contribución

Step-by-step explanation:

De la pregunta, se nos dice:

Dinero total requerido: $ 72,000

El primer amigo contribuyó: 1/3 si el dinero era necesario

Por lo tanto, el primer amigo contribuyó:

1/3 × $ 72 000 = $ 24 000

El segundo amigo contribuyó con 5/6 del resto

La fracción que representa el resto = 1 - 1/3 = 2/3

Por lo tanto, el segundo amigo contribuyó = 5/6 de 2/3 = 5/6 × 2/3 = 5/9

La fracción de la cantidad de dinero que contribuyó el segundo amigo = 5/9

Por lo tanto, la cantidad que contribuyó el segundo amigo =

5/9 × $ 72 000 = $ 40 000.

a) ¿cuanto dinero aporto el tercer amigo?

Esto se calcula como:

$ 72,000 - ($ 24,000 + $ 40,000)

$ 72 000 - $ 64 000

= $ 8,000

Por lo tanto, el tercer amigo aportó $ 8,000.

b) ¿Cual de los tres hizo el mayor aporte?

Del cálculo anterior:

El primer amigo = $ 24,000

Segundo amigo = $ 40,000

Tercer amigo = $ 8,000

Por tanto, el segundo amigo hizo la mayor contribución.

c) ¿cuanto mas puso en relación con que el que aporto menos?

Del cálculo anterior:

El primer amigo = $ 24,000

Segundo amigo = $ 40,000

Tercer amigo = $ 8,000

Por lo tanto, el tercer amigo hizo la menor contribución.

PlZ HELP NOW ASAP!!!

Answers

Answer:

x=33

Step-by-step explanation:

x for ?

sinx=opposite/hypotenuse

sinx=162.5/298=0.54530201

convert to degrees using arcsin(0.54530201)= 33.043

33 degrees rounded to the nearest ones

Other Questions
Explain what a limiting reactant is and describe the process to determine the limiting reactant in the example listed below: Given 3.4 grams of x compound with a molar mass of 85 g and 4.2 grams of y compound with a molar mass of 48 g How much of compound xy can be generated 2x + y2 = 2xy 1. 3x + 6y = 3 and 7x + 3y = 7ons for bo Picasso was the gateway to true modern art. His painting Les Demoiselles d'Avignon is considered the birth of modern art, which influenced many of the other great modern painters. Art doesn't need to be perfectly and accurately represented images to be beautiful; art is intended to invoke a mood to the viewer.Do you agree with the author regarding Picasso, the painting, and the definition of art? Why or why not? Read and choose the correct option. If you love sharks and want to see them in fresh water, you need to go to ___________. Nicaragua El Salvador Mxico Costa Rica Your car must have two red stoplights, seen from ______ feet in the daytime, that must come on when the foot brake is pressed.A. 100B. 200C. 300D. 400 {(-2,8),(4,6),(10,4)} Which point, when added to the set, would form a relation that is not a function? Sheridan Company, had 837000 shares of common stock issued and outstanding at December 31, 2017. On July 1, 2018, an additional 40600 shares of common stock were issued for cash. Sheridan also had unexercised stock options to purchase 31400 shares of common stock at $15 per share outstanding at the beginning and end of 2018. The average market price of Sheridan's common stock was $20 during 2018. What is the number of shares that should be used in computing diluted earnings per share for the year ended December 31, 2018 Calculate the [H+] and pH of a 0.0010 M acetic acid solution. The Ka of acetic acid is 1.76105. Use the method of successive approximations in your calculations. he sum of two nonnegative numbers is 300. What is the maximum value of the product of these two numbers? Identify the sentence that has the subordinating conjunction underlined and is punctuated correctly.A. Please take a few minutes to help me clean up, even if you feel you are too busy.B. We have to finish our work, before we can play a game.C. Although it is not fun to do chores everyone must do their share.D. You must finish your homework early because we have a busy night. Which of the following is true? Tangent is positive in Quadrant I. Sine is negative in Quadrant II. Cosine is positive in Quadrant III. Sine is positive in Quadrant IV. Solve x/5 - 1/2 = x/6 (make sure to type the number only) Can someone help me solve parts (a) and (c) please? Thank you! What are the coordinates of the vertices of the polygon in the graph that are in Quadrant II? A) (4,2) B) (4,3), (0,5), (0,1) C) (5,2), (3,2), (3,4) D) (1,0), (5,2), (3,2), (3,4), (0,5), (0,1) Why is the water colder on the southwest coast of the United States, yet the northeast coast has warmer water? If x3 + ax2 bx + 10 is divisible by x2 3x + 2,find the values of1) a-b2) 2a-b An air-conditioner which uses R-134a operates on the ideal vapor compression refrigeration cycle with a given compressor efficiency.--Given Values--Evaporator Temperature: T1 (C) = 9Condenser Temperature: T3 (C) = 39Mass flow rate of refrigerant: mdot (kg/s) = 0.027Compressor Efficiency: nc (%) = 90a) Determine the specific enthalpy (kJ/kg) at the compressor inlet. Your Answer =b) Determine the specific entropy (kJ/kg-K) at the compressor inlet Your Answer =c) Determine the specific enthalpy (kJ/kg) at the compressor exit Your Answer =d) Determine the specific enthalpy (kJ/kg) at the condenser exit. Your Answer =e) Determine the specific enthalpy (kJ/kg) at the evaporator inlet. Your Answer =f) Determine the coefficient of performance for the system. Your Answer =g) Determine the cooling capacity (kW) of the system. Your Answer =h) Determine the power input (kW)to the compressor. Your Answer = A risk-free, zero-coupon bond has 15 years to maturity. Which of the following is closest to the price per $1,000 of face value at which the bond will trade if the current YTM is 6.1%? a $411.40 b. $553.15 c $663.78 d. $885.05 e. $774.42 An atom of 120In has a mass of 119.907890 amu. Calculate the mass defect (deficit) in amu/atom. Use the masses: mass of 1H atom Simply this question and get marked branlist